Your classmate simplified the following rational expression and asked you if you got the same answer. Determine whether the answer below is correct. If it is not correct, simplify the expression. Show all of your work.


Rational expression: (x^(2)-1) / (x^(2)-4x-5) ÷ (2x^(2)+4x) / (6x^(2)-30x)


Your friend's answer: (2x(x-1)(x+2)) / (6x(x-5)(x-5))

Answers

Answer 1

Answer:

The friend's answer is incorrect.

The correct answer is:

[tex] \dfrac{3(x - 1)}{x + 2} [/tex]

Step-by-step explanation:

[tex] \dfrac{x^2 - 1}{x^2 - 4x - 5} \div \dfrac{2x^2 + 4x}{6x^2 - 30x} = [/tex]

To divide by a fraction, multiply by its reciprocal.

[tex] = \dfrac{x^2 - 1}{x^2 - 4x - 5} \times \dfrac{6x^2 - 30x}{2x^2 + 4x} [/tex]

Multiply the numerators together. Multiply the denominators together.

[tex] = \dfrac{(x^2 - 1)(6x^2 - 30x)}{(x^2 - 4x - 5)(2x^2 + 4x)} [/tex]

Factor every polynomial.

[tex] = \dfrac{(x - 1)(x + 1)(6x)(x - 5)}{(x - 5)(x + 1)(2x)(x + 2)} [/tex]

Divide the numerator and denominator by the common terms.

[tex] = \dfrac{(x - 1)(6)}{(2)(x + 2)} [/tex]

[tex] = \dfrac{3(x - 1)}{x + 2} [/tex]

The friend's answer is incorrect.

The correct answer is:

[tex] \dfrac{3(x - 1)}{x + 2} [/tex]


Related Questions

find the sum of (x²+3xy+y²)+(x³+3x²y+2xy²+y³)​

Answers

x
3
+
3
x
2
y
+
2
x
y
2
+
y
3
+
x
2
+
3
x
y
+
y
2

PLEASE HELP ASAP 30 POINTS

Answers

Answer:

I don't know how to do please let me I will try solve the question

Mọi người giúp em với

Answers

Answer:

bka bla bla bla sorry I newbie

[ INDICES]- Simplify :
1. [tex] \large{ \tt{\frac{ {13}^{ \: 2x + 1} - 5 \times {169}^{x} }{9 \times {169}^{x} } }}[/tex] [ Ans : 2 ]

2. [tex] \large{ \tt{ \frac{ {9}^{ \: n + 2} + 10 \times {9}^{n} }{ {9}^{n + 1} \times 11 - 8 \times {9}^{n} }}}[/tex] [ Ans : 1 ]

- Please show your workings! :)

Answers

Step-by-step explanation:

Hey there!

Please see attached picture for your answer!

Hope it helps!

Answer is in the attachment.

note:

make a slight change in question 1;

sin pi/3 __ __ pi/6 = 1/2(sin pi/2 + sin pi/6)

I think I’m just supposed to fill in the blank? (question off of a p e x) please give explanation!

Answers

Notice that

π/2 = π/3 + π/6

π/6 = π/3 - π/6

Recall the angle sum identities for sine:

sin(x + y) = sin(x) cos(y) + cos(x) sin(y)

sin(x - y) = sin(x) cos(y) - cos(x) sin(y)

By adding these together, we get

sin(x + y) + sin(x - y) = 2 sin(x) cos(y)

==>   sin(x) cos(y) = 1/2 (sin(x + y) + sin(x - y))

Now take x = π/3 and y = π/6 :

sin(π/3) cos(π/6) = 1/2 (sin(π/2) + sin(π/6))

So the blank should be filled with cos.

Solve using the Pythagorean identity

Answers

[tex]\bold{Sin\theta_{1}=-\frac{4}{5}}[/tex]

Answer:

Solution given

Cos[tex]\displaystyle \theta_{1}=\frac{3}{5}[/tex]

consider Pythagorean theorem

[tex]\bold{Sin²\theta+Cos²\theta=1}[/tex]

Subtracting [tex]Cos²\theta[/tex]both side

[tex]\displaystyle Sin²\theta=1-Cos²\theta[/tex]

doing square root on both side we get

[tex]Sin\theta=\sqrt{1-Cos²\theta}[/tex]

Similarly

[tex]Sin\theta_{1}=\sqrt{1-Cos²\theta_{1}}[/tex]

Substituting value of [tex]Cos\theta_{1}[/tex]

we get

[tex]Sin\theta_{1}=\sqrt{1-(\frac{3}{5})²}[/tex]

Solving numerical

[tex]Sin\theta_{1}=\sqrt{1-(\frac{9}{25})}[/tex]

[tex]Sin\theta_{1}=\sqrt{\frac{16}{225}}[/tex]

[tex]Sin\theta_{1}=\frac{\sqrt{2*2*2*2}}{\sqrt{5*5}}[/tex]

[tex]Sin\theta_{1}=\frac{4}{5}[/tex]

Since

In IVquadrant sin angle is negative

[tex]\bold{Sin\theta_{1}=-\frac{4}{5}}[/tex]

Answer:

[tex]\sin(\theta_1)=-\frac{4}{5}[/tex]

Step-by-step explanation:

We'll use the Pythagorean Identity [tex]\cos^2(\theta)+\sin^2(\theta)=1[/tex] to solve this problem.

Subtract [tex]\cos^2(\theta)[/tex] from both sides to isolate [tex]\sin^2(\theta)[/tex]:

[tex]\sin^2(\theta)=1-\cos^2(\theta)[/tex]

Substitute [tex]\cos(\theta)=\frac{3}{5}[/tex] as given in the problem:

[tex]\sin^2(\theta_1)=1-(\frac{3}{5}^2)[/tex]

Simplify:

[tex]\sin^2\theta_1=1-\frac{9}{25}[/tex]

Combine like terms:

[tex]\sin^2\theta_1=\frac{16}{25}[/tex]

For [tex]a^2=b[/tex], we have two solutions [tex]a=\pm \sqrt{b}[/tex]:

[tex]\sin\theta_1=\pm \sqrt{\frac{16}{25}},\\\begin{cases}\sin \theta_1=\frac{4}{5},\\\sin \theta_1=\boxed{-\frac{4}{5}}\end{cases}[/tex]

Since the sine of all angles in quadrant four return a negative output, [tex]\frac{4}{5}[/tex] is extraneous and our answer is [tex]\boxed{\sin(\theta_1)=-\frac{4}{5}}[/tex]

The graph shows the function f(x) = 2x
What is the value of x when fx) = 8?

Answers

Answer:

4 = x

Step-by-step explanation:

f(x) =2x

Let f(x) = 8

8 =2x

Divide each side by 2

8/2 = 2x/2

4 = x

Answer:

4

Step-by-step explanation:

f(x) = 2x

When f(x) = 8, x = 8/2 = 4.

Hope this helped,

~cloud

I only need the answer

Answers

Answer:

1

Step-by-step explanation:

The given equation of the function is y = -a·(x - h)² + 1

The positive constants of the equation = a, and h

The points the function crosses the x-axis = 2, and 4

Where the function crosses the x-axis, y = 0, and x = 2, and 4, therefore, when x = 2, we have;

y = 0 = -a·(2 - h)² + 1

When x = 4, we have;

0 = -a·(4 - h)² + 1

-a·(2 - h)² + 1 = -a·(4 - h)² + 1

-a·(2 - h)² = -a·(4 - h)²

(2 - h)² = (4 - h)²

±(2 - h) = +#±(4 - h)

When

(2 - h) is negative, and (4 - h) is positive, but the same magnitude, we have';

-(2 - h) = +(4 - h)

2·h = 4 + 2 = 6

h = 3

0 = -a·(4 - h)² + 1 = -a·(4 - 3)² + 1 = -a + 1

Therefore, a = 1

Can someone help me with this math homework please!

Answers

Answer:

Step-by-step explanation:

Eight years ago, the daughters age was thrice the son's age. Now the daughter's age is 4 years more than the son's age. Find their present ages.​

Answers

Answer:

Let s be the son’s current age and d be the daughter’s current age. The system of equations is:

s - 10 = 2(d - 10)

s = 3 + d

Since s is already set to an equation, we can use the substitution method for s in the other equation:

s = 3 + d

s - 10 = 2(d - 10)

3 + d - 10 = 2(d - 10)

Simplify and solve for d:

3 + d - 10 = 2(d - 10)

-7 + d = 2d - 20

-7 = d - 20

13 = d

The daughter is 13 years old. To solve for the son’s age, we will plug in the solution for d into one of the equations. The second one is simpler so we will use that:

s = 3 + d

s = 3 + 13

s = 16

The son is 16 years old. Let us use the other equation to check our solutions:

s - 10 = 2(d - 10)

16 - 10 = 2(13 - 10)

6 = 2(3)

6 = 6

It checks out. The son is 16 years old, and the daughter is 13 years old.

The present age of the daughter and son are 14 and 10 years respectively.

Let the age of the daughter be x

Let the age of the son be y

If the daughter's age is 4 years more than the son's age now, then,

x = y + 4 ............. 1

If Eight years ago, the daughters' age was thrice the son's age, then;

Daughter = x - 8

Son  = y - 8

Hence, x - 8 =3(y - 8).................. 2

Substitute equation 1 into 2 to have:

x - 8 =3(y - 8).

y + 4 - 8 = 3(y - 8)

y - 4 = 3y - 24

y - 3y = -20

-2y = -20

y = 10

Recall that x = y + 4

x = 10 + 4

x = 14

Hence the present age of the daughter and son are 14 and 10 years respectively.

LEarn more here: https://brainly.com/question/16510024

Scarlett made a profit of $250.00 with her mobile car wash company

Answers

Not enough information to solve..... Please make your question more clear

Which system of linear inequalities is graphed?

Answers

Answer:

The first one.

Step-by-step explanation:

Graph lines as if the inequalities were equal signs.

X = -3 is a vertical line at x = -3, because it's less than we shade to the left. All numbers less than -3 are to the left. The line is dashed because there is no equal to. Only less than. The line is not included in the solution set.

y = -x - 1 is a line with a y-intercept of -1 and a slope of -1. All values that are less that y are below the line. Because it's less than or equal to the line is solid.

Answer:

A

Step-by-step explanation:

The vertical line  is dotted at -3 and shaded to the left

x < -3

This gives us two choices left

A and C

The other line has a y intercept at -1 and is solid and shaded to the left

It is of the form

y ≤ mx+1  

We know the slope is negative since is goes down from left to right

The only Choice is A

Can someone help me with this math homework please!

Answers

Answer:

(-3,-6) and (-3,2)

Step-by-step explanation:

A line with an undefined slope is vertical. In this case it must have x coordinates equal to -3

1 Which one of the following expression represents the sum of the expressions (5x - 13xy + 14y) and (12xy - 6x - 12y)?​

Answers

Answer:

(12xy-6x -12y)

12xy-18xy

6xy

Answer:

- xy - x + 2y

Step-by-step explanation:

5x - 13xy + 14y + 12xy - 6x - 12y ← collect like terms

= (- 13xy + 12xy) + (5x - 6x) + (14y - 12y)

= - xy - x + 2y

sec²x + cosec²x ≡sec²xcosec²x​

proving qn pls i nd it by tdy

Answers

Step-by-step explanation:

sec²x + cosec²x = sec²x.cosec²x

1/cos²x + 1/sin²x = 1/cos²x.1/sin²x

or, (sin²x+cos²x)/sin²x.cos²x = 1/(sin²x.cos²x)

or, 1/(sin²x.cos²x) = 1/(sin²x.cos²x)

Hence,

sec²x + cosec²x = sec²x.cosec²x proved!!

Doanh nghiệp bán trả góp 1 bất động sản có giá thanh toán là 32.000trđ . Thu điều cả vốn lẫn lãi trong 10 năm với lãi suất trả chậm là 10% / năm . Xác định số vốn phải thu ở năm thứ 6 ?

Answers

Answer:

Step-by-step explanation:

If x=3 ,y=4 than what is the value?

Answers

I am pretty sure it is 91

Find the value of x that will make L||M.
6x + 8
4x + 2
X =[?]

Answers

Answer:

6x+8+4x+2=180

so!! 10x+10=180

10x=170

x=17

Answer:

[tex]x=17[/tex]

Step-by-step explanation:

The two angles labelled [tex]6x+8[/tex] and [tex]4x+2[/tex] are co-interior angles. When two parallel lines are cut by a traversal, co-interior angles are supplementary, meaning they add up to 180 degrees. Therefore, if line L is parallel to line M, [tex]6x+8[/tex] and [tex]4x+2[/tex] must be supplementary:

[tex]6x+8+4x+2=180[/tex]

Combine like terms:

[tex]10x+10=180[/tex]

Subtract 10 from both sides:

[tex]10x=170[/tex]

Divide both sides by 10:

[tex]x=\frac{170}{10}=\boxed{17}[/tex]

Instructions: Point R is the centroid. Find DU if DR = 14.

Answers

Answer:

21

Step-by-step explanation:

If DR measures 14, DU will have to measure DR + half the measure of DR

DU = 14 + 7

Du = 21

Any close answers to this will be correct!

If Tanya cleans a 1500 square foot building for a total of $107.00, what is the amount Tanya gets paid per square foot?

Answers

Answer:

Tanya will get approximately $0.07 per square foot.

This can be solved by creating a ratio between the amount she is paid and the amount of square feet she cleans.

(1500)/(107)=(1)/(x) = 0.0713

Answer:

0.0713

Step-by-step explanation:

Tanya will get approximately $0.07 per square foot.

This can be solved by creating a ratio between the amount she is paid and the amount of square feet she cleans.

(1500)/(107)=(1)/(x) = 0.0713

Find the distance between the points (-2, -10) and (-9,-5) on a coordinate plane.

Answers

Answer:

√74

Step-by-step explanation:

I'm not sure how to answer this!!

Answers

Answer:

I think it's a

Step-by-step explanation:

hsjshbwibs hshhsvsu hshsbsbus

Good Afternoon. I hope you’re well:) I need to understand how to go about solving this without a calculator, please. Thank you so much and have a blessed day!

Answers

Answer:

Step-by-step explanation:

Being able to do this requires that you understand what is being asked and that you understand domain. This is a piecewise function, made up of 4 different parts, and each part has a domain different from every other part. Now look at the solutions. We are given coordinates, x and y. So this is how this problem is done.

In a. the coordinate is (-1, 1), right? x = -1 here, so the equation that we "pick" (I'll explain that in a sec) has to have a domain where -1 will fall. The first part of this piecewise has a domain of less than or equal to -11. Is -1 less than or equal to -11? No, it is not. -1 is greater than -11, so the equation we "pick" to use will not be this one. Look at the next piece of the function and note its domain. This domain is that x is greater than -11 and less than 5. Does -1 fit in that domain? Is -1 included in that spread of numbers? Yes it is, so that is the equation we will use. By use I mean that we will plug in -1 for x and see if y = 1 (that number comes from the coordinate (-1, 1) where y = 1). The equation is y = x + 2 and plugging in -1 for x:

y = -1 + 2 so

y = 1 and this point is on the piecewise. Let's do one more example so you can see how it looks when it DOESN'T work out, ok?

Look at b. The coordinate is (-2, -10). x = -2, so the same domain, same equation: x + 2. We plug in -2 for x to find y:

y = -2 + 2 so

y = 0.  0 does not equal -10, so this point is not on that graph.

The key here is picking the equation whose domain includes your x value and evaluating the equation at that value of x to find y.

Please help me! I cant figure this out

Answers

AB is the same as A multiplied by B. In order to get this value, we need to figure out the value of A and B.

We have two equations here:

a + 5b = 2b + 13

a = 3b - 5

The second equation already has A isolated. So, we can take that equation and plug in the value for A. Doing so will allow us to solve for B.

(3b - 5) + 5b = 2b + 13

8b - 5 = 2b + 13

6b - 5 = 13

6b = 18

b = 3

Now that we know B, we can use our equation that isolated A.

a = 3(3) - 5

a = 9 - 5

a = 4

Therefore, a = 4 and b = 3, and ab = 12.

Hope this helps!

Maya is 14 years old. Her brother Jorge is 3 years more than half her age. Which of the following is the correct expression for Jorge's age?

(7 x 2) + 3
7 − (2 + 3)
(14 ÷ 2) + 3
14 ÷ (7 − 3)

Answers

Answer:

3rd Part

(14÷2) + 3

Step-by-step explanation:

The 3rd part is the answer and on solving the equation, 10 comes as the answer.

Answer:

(14 ÷ 2) + 3

Step-by-step explanation:

Graph the function.
is it a, b, c or d?

Answers

Answer:

D

Step-by-step explanation:

Equation of line is y=2x-2

1. Take a look at the steps below for solving the
equation 4(3 - 2x) = -20.
Step 1: 4(3 - 2x) = -20
Step 2: 12 - 8x = -20
Step 3: 12-8x - 12 = -20 - 12
Step 4: - 8x + 0 = -32
Step 5:
- 8x = -32
1
Step 6. - 8x
-1
8
8
Step 7:
x = 4
1
= -32
Which property justifies the work between ...
A. Step 1 and Step 2?
B. Step 2 and Step 3?
C. Step 3 and Step 4?
D. Step 4 and Step 5?
E. Step 5 and Step 6?
F. Step 6 and Step 7?

Answers

Answer:

x= 1

Step-by-step explanation:

Work is shown in the picture

Helpppp
Graph the quadratic function y 9x Ox 140. What are the solutions of the quadratic equation 0 - 0 - 6x
140?

Answers

Answer:

                  Answer:bf                  

Step-by-step explanation:

Please help! The answer isn’t 36!
Find the value of the expression:

Answers

Answer:

3.6

Step-by-step explanation:

Hi there!

We are given this expression:

.6√36 (.6*√36)

And we want to find the value of it

.6 can be re-written as 0.6

In that case,

0.6*√36

First, simplify what's under the radical: √36, which is equal to 6 (6*6=36)

The expression then becomes:

0.6*6

Multiply those numbers together

0.6*6=3.6

Hope this helps!

Which of the following statements follows from (x - 3)2 = 7?

Answers

Answer:

x = 17

Step-by-step explanation:

(x-3)2 = 7

Divide both sides by two

(x-3)2 = 7

2 2

x-3= 31/2

x= 31/2-3

x= 1/2 or 0.5

(x-3) 2= 7
x-3 = 7/2
x = (7/2) × 3
x = 21/2 or 10.5
Other Questions
Sam's monthly bills are normally distributed with mean 2700 and standard deviation 230.9. He receives two paychecks of $1500 each in a month, post taxes and withholdings. What is the probability that his expenses will exceed his income in the following month?) 10%. B) 16%.C) 21%.D) 29%.E) 37%. Answer plssssssss!!!!!! You've learned about some of the benefits of a warm-up. How might rushing into a sport like basketball without a warm-up affect you negatively? Explain. A car travels 60 kilometers in one hour before a piston breaks, then travels at 30 kilometers per hour for the remaining 60 kilometers to its destination. What is its average speed in kilometers per hour for the entire trip? Suppose that the value of a stock varies each day from $12.82 to $28.17 with a uniform distribution.Find the third quartile; 75% of all days the stock is below what value? (Enter your answer to the nearest cent.) How many molecules (or formula units) are in 138.56 g C4H10 Express your answer using four significant figures. importance of settlement which of the following are exterior angles? how can natural disasters affect the life of people if in case of a pandemic What is the simplest version of 9/164/18 Argue to the view that Vietnam will face the low birthrate and aging population in 2050 that South Korea is experiencing. A random sample of 25 values is drawn from a mound-shaped and symmetric distribution. The sample mean is 9 and the sample standard deviation is 2. Use a level of significance of 0.05 to conduct a two-tailed test of the claim that the population mean is 8.5.(a) Is it appropriate to use a Student's t distribution? Explain.Yes, because the x distribution is mound-shaped and symmetric and is unknown.No, the x distribution is skewed left. No, the x distribution is skewed right.No, the x distribution is not symmetric.No, is known.How many degrees of freedom do we use?(b) What are the hypotheses?H0: = 8.5; H1: > 8.5H0: = 8.5; H1: 8.5 H0: = 8.5; H1: < 8.5H0: < 8.5; H1: = 8.5H0: > 8.5; H1: = 8.5(c) Compute the t value of the sample test statistic. (Round your answer to three decimal places.)t =(d) Estimate the P-value for the test.P-value > 0.2500.100 < P-value < 0.250 0.050 < P-value < 0.1000.010 < P-value < 0.050P-value < 0.010(e) Do we reject or fail to reject H0?At the = 0.05 level, we reject the null hypothesis and conclude the data are statistically significant.At the = 0.05 level, we reject the null hypothesis and conclude the data are not statistically significant. At the = 0.05 level, we fail to reject the null hypothesis and conclude the data are statistically significant.At the = 0.05 level, we fail to reject the null hypothesis and conclude the data are not statistically significant.(f) Interpret the results.There is sufficient evidence at the 0.05 level to reject the null hypothesis.There is insufficient evidence at the 0.05 level to reject the null hypothesis. How much BaSO4 can be formed from 196.0 g of H2SO4? A 3.00 kg box sliding west at 2.00 m/s makes an inelastic collision with a second box sliding 1.50 m/s east. Afterwards, they both come to a stop. What was the mass of the second box? USING THE ONTARIO BUILDING CODE TO SOLVE THESE QUESTIONS ARCH1013Architectural Engineering1. Calculate the minimum width and thickness of footings required to support walls of Cassidy House Elevation B. (1.5/4). a)Footing width and thickness of exterior walls, if the floor joists span less than 4.9 m:Cassidy is a Two-storey house, minimum thickness of footing (Table 9.15.3.4.)= Two storeys of Brick veneer (9.15.3.5.) = Adjusted Footing width for Cassidy House = Thickness (9.15.3.8.)= b)Assume that the house has a load bearing masonry interior wall that is 150mm thick supporting both first and second storey. Determine the minimum width and thickness required of its footing:Min. thickness of footing for an interior wall that supports two floors (Table 9.15.3.4) = Adjust for two storey of masonry wall (9.15.3.6.) = Thickness = 2. Answer the following questions based on foundations details of Cassidy Elevation B. (1.5/4).a)What is the maximum height of finished ground the foundation wall can support, if it is 2.5m high?Table 9.15.4.2.A, Construction Notesb) If you were to build this wall with 190mm thick ICF, what is the minimum vertical reinforcement required, using the height of foundation wall and ground found in earlier questions?Table 9.15.4.B.c) If you were to build this wall with unreinforced concrete blocks, what is the minimum thickness of the foundation wall?Table 9.15.4.2.A.3. Calculate the footing width of an external wall of a three storey wood frame brick veneered house. Assume it supports joists spanning 11.5m on first and second floors and 10m on third floor. (1/4)9.15.2.4.B: use the formula; W = w (sjs/ storeys x 4.9)If anyone has any Idea on how to do this or has any Architectural background please explain to me! Elevation B: Page 13 The square root of 0.25 is 0.5 which is a greater number. Give another number whose square root is larger than the number and explain why. Explain why body size and waist circumference is an accurate measurement for allethnicities in determining overall risk for heart disease or diabetes. HELPPPP Ice Cream Corporation uses the weighted-average method in its process costing system. Data concerning the first processing department for the most recent month are listed below: Beginning work in process inventory: Units in beginning work in process inventory 2,400 Materials costs $ 14,500 Conversion costs $ 6,600 Percent complete with respect to materials 75 % Percent complete with respect to conversion 20 % Units started into production during the month 11,100 Units completed and transferred to the next department during the month 10,000 Materials costs added during the month $ 173,600 Conversion costs added during the month $ 243,600 Ending work in process inventory: Units in ending work in process inventory 3,500 Percent complete with respect to materials 90 % Percent complete with respect to conversion 30 % The cost per equivalent unit for materials for the month in the first processing department is closest to: Water, H2O ,and methane ,CH4, are both covalent substances . why is water a liquid at room temperature while methane is a gas ? A. Carbon compounds tend to be gases . B. Water has a stronger intermolecular forces Find the value of x.A. 99B. 9C. 90D. 11ILL GIVE BRAINLIEST